Подтвердить что ты не робот

Временная сложность программы с использованием уравнения рекуррентности

Я хочу узнать временную сложность программы с использованием рекуррентных уравнений. То есть..

int f(int x)
{
if(x<1) return 1;
 else  return f(x-1)+g(x); 
}
int g(int x)
{
if(x<2) return 1;
 else return f(x-1)+g(x/2);
}

Я пишу его рекуррентное уравнение и попытался его решить, но он продолжает усложнять

T(n) =T(n-1)+g(n)+c
         =T(n-2)+g(n-1)+g(n)+c+c
         =T(n-3)+g(n-2)+g(n-1)+g(n)+c+c+c
         =T(n-4)+g(n-3)+g(n-2)+g(n-1)+g(n)+c+c+c+c
         ……………………….
        ……………………..
        Kth time …..
        =kc+g(n)+g(n-1)+g(n-3)+g(n-4).. .. . … +T(n-k)

Let at kth time input become 1
Then n-k=1
         K=n-1
Now i end up with this..
T(n)= (n-1)c+g(n)+g(n-1)+g(n-2)+g(n-3)+….. .. g(1)

Я не могу его решить дальше. В любом случае, если мы посчитаем количество вызовов функций в этой программе, можно легко увидеть, что временная сложность экспоненциальна, но я хочу доказать ее с помощью повторения. как это можно сделать?

enter image description here

Объяснение в Anwer 1, выглядит правильно, аналогичная работа я сделал.

Самая сложная задача в этом коде - написать уравнение рекурсии. Я нарисовал еще одну диаграмму, я определил некоторые шаблоны, я думаю, что мы можем получить некоторую помощь в этой диаграмме, что могло бы быть возможным рекуррентным уравнением.

For f(2)

For f(3)

And I came up with this equation , not sure if it is right ??? Please help.

T(n) = 2*T(n-1) + c * logn
4b9b3361

Ответ 1

Хорошо, мне кажется, что я смог доказать, что f(x) = Theta(2^x) (обратите внимание, что временная сложность такая же). Это также доказывает, что g(x) = Theta(2^x) как f(x) > g(x) > f(x-1).

Сначала, как отметили все, легко доказать, что f(x) = Omega(2^x).

Теперь мы имеем отношение, что f(x) <= 2 f(x-1) + f(x/2) (так как f(x) > g(x))

Покажем, что при достаточно больших x существует некоторая константа K > 0 такая, что

f(x) <= K*H(x), where H(x) = (2 + 1/x)^x

Отсюда следует, что f(x) = Theta(2^x), как H(x) = Theta(2^x), что само следует из того факта, что H(x)/2^x -> sqrt(e) as x-> infinity (wolfram alpha ссылка предел).

Теперь ( предупреждение: более тяжелая математика, perhap cs.stackexchange или math.stackexchange лучше подходит)

в соответствии с wolfram alpha (щелкните ссылку и посмотрите расширение серии рядом с x = бесконечность),

H(x) = exp(x ln(2) + 1/2 + O(1/x))

И опять же, согласно wolfram alpha (нажмите ссылку (отличную от предыдущей) и посмотрите разложение серии для x = бесконечность), мы что

H(x) - 2H(x-1) = [1/2x + O(1/x^2)]exp(x ln(2) + 1/2 + O(1/x))

и поэтому

[H(x) - 2H(x-1)]/H(x/2) -> infinity as x -> infinity

Таким образом, при достаточно больших x (скажем, x > L) имеет место неравенство

H(x) >= 2H(x-1) + H(x/2)

Теперь существует некоторая K (зависящая только от L (например, K = f (2L))) такая, что

f(x) <= K*H(x) for all x <= 2L

Теперь мы продолжим (сильную) индукцию (вы можете вернуться к натуральным числам, если хотите)

f(x+1) <= 2f(x) + f((x+1)/2)

По индукции правая часть

<= 2*K*H(x) + K*H((x+1)/2)

И мы ранее доказали, что

2*H(x) + H((x+1)/2) <= H(x+1)

Таким образом, f(x+1) <= K * H(x+1)

Ответ 2

Используя memoisation, обе функции могут быть легко вычислены в O (n) времени. Но программа занимает не менее O (2 ^ n) времени и, следовательно, является очень неэффективным способом вычисления f(n) и g(n)

Чтобы доказать, что программа принимает не более O (2 + epsilon) ^ n время для любого epsilon > 0:

Пусть F (n) и G (n) - число вызовов функций, которые выполняются при оценке f (n) и g (n) соответственно. Ясно (подсчет добавления как 1 вызов функции):

F (0) = 1; F (n) = F (n-1) + G (n) + 1 G (1) = 1; G (n) = F (n-1) + G (n/2) + 1

Тогда можно доказать:

  • F и G монотонны
  • F > G
  • Определить H (1) = 2; H (n) = 2 * H (n-1) + H (n/2) + 1
  • ясно, H > F
  • для всех n, H (n) > 2 * H (n-1)
  • поэтому H (n/2)/H (n-1) → 0 при достаточно больших n
  • следовательно, H (n) (2 + epsilon) * H (n-1) для всех epsilon > 0 и достаточно больших n
  • поэтому H в O ((2 + epsilon) ^ n) для любого epsilon > 0
  • (Edit: изначально я пришел к выводу, что верхняя граница O (2 ^ n). Это неверно, как указал nhahtdh, но см. ниже)
  • так что это лучшее, что я могу доказать.... Поскольку G < F < H они также находятся в O ((2 + epsilon) ^ n) для любого epsilon > 0

Постскриптум (после просмотра решения г-на Кноукса): Поскольку i.m.h.o хорошее математическое доказательство дает представление, а не множество формул, и SO существует для всех этих будущих поколений (привет!):

Для многих алгоритмов вычисление f (n + 1) включает в себя дважды (трижды,...) объем работы для f (n), плюс нечто большее. Если это больше становится относительно меньше с ростом n (что часто бывает), используя фиксированный epsilon, как указано выше, не является оптимальным. Замена эпсилона выше некоторой убывающей функцией ε (n) of n будет во многих случаях (если ε будет достаточно быстро убывать, скажем, ε (n) = 1/n), получим верхнюю грань O ((2 + ε (n)) ^ n) = O (2 ^ n)

Ответ 3

Пусть f (0) = 0 и g (0) = 0

Из функции мы имеем

f(x) = f(x - 1) + g(x) 
g(x) = f(x - 1) + g(x/2)

Подставляя g (x) в f (x), получим:

f(x) = f(x-1) + f(x -1) + g(x/2)

& there4; f (x) = 2f (x-1) + g (x/2)

Разлагая это, получим:

f(x) = 2f(x-1)+f(x/2-1)+f(x/4-1)+ ... + f(1)

Пусть s (x) - функция, определенная следующим образом:

s(x) = 2s(x-1)

Теперь ясно, что f (x) = & Omega; (s (x)).

Сложность s (x) равна O (2 x).

Поэтому функция f (x) = & Omega; (2 x).

Ответ 4

Я думаю, ясно, что f (n) > 2 n так как f (n) > h (n) = 2h (n-1) = 2 n.

Теперь я утверждаю, что для каждого n существует & epsilon; такой, что: f (n) (2 + & epsilon;) n, чтобы увидеть это, позвольте сделать это по индукции, но чтобы сделать его более разумным сначала, я буду использовать & epsilon; = 1, чтобы показать f (n) <= 3 n тогда я продолжу его.

Мы будем использовать сильную индукцию, пусть для каждого m < n, f (m) 3 m то имеем:

f(n) = 2[f(n-1) + f(n/2 -1) + f(n/4 -1)+ ... +f(1-1)]

но для этой части:

A = f(n/2 -1) + f(n/4 -1)+ ... +f(1-1)

имеем:

f(n/2) = 2[f(n/2 -1) + f(n/4 -1)+ ... +f(1-1]) ==>

A <= f(n/2)   [1]

Итак, мы можем переписать f (n):

f(n) = 2f(n-1) + A < 2f(n-1) +f(n/2),

Теперь вернемся к нашему утверждению:

f(n) < 2*3^(n-1) + 2*3^(n/2)==>
f(n) < 2*3^(n-1) + 3^(n-1) ==>
f(n) < 3^n.  [2]

В силу [2] доказательство f (n) & in; O (3 n) завершено.

Но если вы хотите распространить это на формат (2 + & epsilon;) n просто используйте 1, чтобы заменить неравенство, то мы будем иметь

для & epsilon; > 1/(2 + & epsilon;) n/2-1 → f (n) (2 + & эпсилон;). п [3]

Также в [3] вы можете сказать, что для каждого n существует & epsilon; такой, что f (n) (2 + & epsilon;) n на самом деле существует постоянный & epsilon; такой, что при n > n 0 f (n) & in; O ((2 + & epsilon;) n). [4]

Теперь мы можем использовать wolfarmalpha как @Knoothe, установив & epsilon; = 1/n, тогда мы получим:

f (n) (2 + 1/n) n что приводит к f (n) e * 2 n а по нашей простой нижней границе в начале мы имеем: f (n) & in; & Theta;. (2 ^ п) [5]

PS: я точно не вычислял epsilon, но вы можете сделать это с помощью ручки и бумаги просто, я думаю, что этот эпсилон не правильный, но его легко найти, и если мне трудно сказать, что это сложно, и я Я напишу его.